Probability of Root Placement

Algebra Level 4

k k is uniformly chosen from the interval [ 5 , 5 ] [ -5, 5] . Let p p be the probability that the quadratic f ( x ) = x 2 + k x + 1 f(x) = x^2 + kx + 1 has both roots between -2 and 4. What is the value of 1000 p \lfloor 1000 p \rfloor ?

Details and assumptions

Greatest Integer Function: x : R Z \lfloor x \rfloor: \mathbb{R} \rightarrow \mathbb{Z} refers to the greatest integer less than or equal to x x . For example 2.3 = 2 \lfloor 2.3 \rfloor = 2 and 3.4 = 4 \lfloor -3.4 \rfloor = -4 .


The answer is 275.

This section requires Javascript.
You are seeing this because something didn't load right. We suggest you, (a) try refreshing the page, (b) enabling javascript if it is disabled on your browser and, finally, (c) loading the non-javascript version of this page . We're sorry about the hassle.

15 solutions

Vamsi P
May 20, 2014

1) For roots to be real k 2 4 0 k^2-4 \geq 0 which gives k 2 k \geq 2 or k 2 k \leq -2

2) Value of f ( x ) f(x) should be positive at x = 2 x=-2 and x = 4 x=4 which gives conditions k 2.5 k \leq 2.5 & k 4.25 k \geq -4.25 respectively.

3) Vertex of parabola must be between -2 and 4, which gives 4 > k > 8 4 > k > -8 .

From the above conditions k k belongs to [ 4.25 , 2 ] [ 2 , 2.5 ] [-4.25,-2]\cup [2,2.5] . Hence, Length of k = 2.75 k =2.75 , which gives p = 2.75 10 p = \frac {2.75}{10} , 1000 p = 275 \lfloor 1000p \rfloor =275 .

[Latex edits - Calvin]

All other solutions tried to approach this problem using the quadratic roots formula, and made numerous mistakes along the way.

Common mistakes.

  1. Claiming that solving k 2 4 8 + k \sqrt{k^2 - 4} \leq 8+k is equivalent to squaring both sides to obtain 4.25 k -4.25 \leq k . This is not true. The solution to the inequality is 17 4 k 2 2 k \frac {17}{4} \leq k \leq -2 \cup 2 \leq k .

  2. Claiming that it is sufficient to solve for the exact values 4 = k + k 2 4 2 k = 5 2 4 = \frac{-k + \sqrt{k^2-4}}{2} \Rightarrow k = \frac{5}{2} without explaining why this would gives the domain that we are interested in.

  3. Not flipping the sign of the inequality when multiplying by 1 -1 .

Calvin Lin Staff - 7 years ago

Log in to reply

In the domain of probability you included -5 and 5 but in solution did not had [ ] brackets in all the range so you lose some no.'s . So probability should be [274.999999999...]=274

Aman Anand - 5 years, 4 months ago

Log in to reply

Nope. The number 5 occurs with 0 probability, and hence whether it's included or not, doesn't affect the probability. In fact, we could remove a countable number of these values, and still end up with the same probability. Check out this problem .

Furthermore, check out is 0.999 = 1 0.999 \ldots = 1 ?

Note: 5 is included but -5 isn't

Calvin Lin Staff - 5 years, 4 months ago

It is given in question" between" so equality does not holds in 2.5 and -17/4.

Aman Anand - 5 years, 4 months ago
Calvin Lin Staff
May 13, 2014

We note that the given function always has a positive leading coefficient, so the quadratic is a parabola that opens upwards. As such, for the quadratic to have roots between -2 and 4, it is necessary and sufficient to satisfy the following 4 conditions:

  1. It must have real roots. Hence Discriminant 0 \mbox{Discriminant} \geq 0 k 2 4 0 k 2 \Rightarrow k^2 -4 \geq 0 \Rightarrow k \leq -2 or k 2 k \geq 2 .

  2. f ( 2 ) 0 f(-2) \geq 0 . Hence 4 2 k + 1 0 k 5 2 \Rightarrow 4 - 2k + 1 \geq 0 \Rightarrow k \leq \frac{5}{2}

  3. f ( 4 ) 0 f(4) \geq 0 . Hence 16 + 4 k + 1 0 k 17 4 \Rightarrow 16 + 4k + 1 \geq 0 \Rightarrow k \geq \frac{-17}{4}

  4. The vertex of the parabola must lie between -2 and 4. Hence 2 k 2 4 8 k 4 -2 \leq \frac {-k} {2} \leq 4 \Rightarrow -8 \leq k \leq 4 .

Putting these together we have two possible intervals for k k : 17 4 k 2 - \frac {17}{4} \leq k \leq -2 and 2 k 5 2 2 \leq k \leq \frac {5}{2} .

Hence, the probability is p = ( 2 ( 17 4 ) ) + ( 5 2 2 ) 5 ( 5 ) = 11 4 10 = . 275 p = \frac{\left(-2 - \left(-\frac{17}{4}\right) \right) + \left(\frac{5}{2} - 2\right)}{5 - (-5) } = \frac {\frac {11}{4}}{10} = .275 . Thus 1000 p = 275 \lfloor 1000p \rfloor = 275 .

Yong See Foo
May 20, 2014

The roots of f ( x ) f(x) by the quadratic formula is k ± k 2 4 2 \frac{-k\pm \sqrt{k^2-4}}{2} . So, we must have 2 k k 2 4 2 4 2 k 2.5 -2\leq \frac{-k-\sqrt{k^2-4}}{2}\leq 4 \Rightarrow 2\leq k\leq 2.5 or k 2 k\leq -2 and 2 k + k 2 4 2 4 2 k -2\leq \frac{-k+ \sqrt{k^2-4}}{2}\leq 4\Rightarrow 2\leq k or 4.25 k 2 -4.25\leq k\leq -2 . Combining these 2 inequalities, we get 4.75 k 2 -4.75\leq k \leq -2 or 2 k 2.5 2\leq k\leq 2.5 . Then p = ( 2.5 2 ) + ( 2 ( 4.25 ) ) 5 ( 5 ) = 2.75 10 p=\frac{(2.5-2)+(-2-(-4.25))}{5-(-5)}=\frac{2.75}{10} . Therefore, 1000 p = 275 \lfloor 1000p\rfloor=275 .

Kevin Sun
May 20, 2014

The two roots of f ( x ) = x 2 + k x + 1 f(x) = x^2 + kx + 1 are k ± k 2 4 2 \frac{-k \pm \sqrt{k^2-4}}{2} . We need these to be real, so k 2 k \ge 2 or k 2 k \le -2 . If they are real, then they must be between 2 -2 and 4 4 , so 2 k ± k 2 4 2 4 -2 \le \frac{-k \pm \sqrt{k^2-4} }{2} \le 4 . Rearranging, we get that 4 + k ± k 2 4 k + 8 -4+k \le \pm \sqrt{k^2-4} \le k+8 . We only need to consider k 4 k 2 4 k-4 \le -\sqrt{k^2-4} and k 2 4 k + 8 \sqrt{k^2-4} \le k+8 , because the other cases are true if these two are true. Solving the first inequality, we get that k 5 2 k \le \frac 52 , and solving the second inequality, we get that k 17 4 k \ge -\frac{17}{4} . Thus x [ 17 4 , 2 ] [ 2 , 5 2 ] x \in [-\frac{17}{4},-2] \cup [2,\frac{5}{2}] , so p = 11 40 p = \frac{11}{40} , so 1000 p = 275 \left \lfloor 1000p \right \rfloor = 275 .

黎 李
May 20, 2014

p=0.275

Zi Song Yeoh
May 20, 2014

From x 2 + k x + 1 = 0 x^{2} + kx + 1 = 0 , we get x = k ± k 2 4 2 x = \frac{-k \pm \sqrt{k^{2} - 4}}{2} . From the roots are between 2 -2 and 4 4 , we conclude that 4 < k ± k 2 4 < 8 -4 < -k \pm \sqrt{k^{2} - 4} < 8 , and therefore k 2 4 > k 4 \sqrt{k^{2} - 4} > k - 4 , k 2 4 > ( k + 8 ) \sqrt{k^{2} - 4} > -(k + 8) , k 2 4 < 4 k \sqrt{k^{2} - 4} < 4 - k and k 2 4 < k + 8 \sqrt{k^{2} - 4} < k + 8 .

Now, consider 4 4 cases.

Case 1 1 : 5 k 2 -5 \le k \le -2

Then k 4 ( k + 8 ) , k + 8 4 k k - 4 \le -(k + 8), k + 8 \le 4 - k . Thus, we just need to consider k 2 4 > ( k + 8 ) , k 2 4 < k + 8 \sqrt{k^{2} - 4} > -(k + 8), \sqrt{k^{2} - 4} < k + 8 . The first inequality clearly holds since in [ 5 , 5 ] [-5, 5] , ( k + 8 ) < 0 k 2 4 -(k + 8) < 0 \le \sqrt{k^{2} - 4} , and the second holds if and only if 17 4 k 2 - \frac{17}{4} \le k \le -2 . Since both sides of the second inequality is nonnegative, we can square both sides and conclude that k 17 4 k \ge - \frac{17}{4} , and thus the conclusion that if 5 k 2 -5 \le k \le -2 , then 17 4 k 2 - \frac{17}{4} \le k \le -2 .

Case 2 2 : 2 < k < 2 -2 < k < 2

Then the discriminant, k 2 4 = ( k + 2 ) ( k 2 ) < 0 x 2 + k x + 1 = 0 k^{2} - 4 = (k + 2)(k - 2) < 0 \Rightarrow x^{2} + kx + 1 = 0 has no real roots, which is a contradiction.

Case 3 3 : 2 k < 4 2 \le k < 4

Since k 4 > ( k + 8 ) , k + 8 > 4 k k - 4 > -(k + 8), k + 8 > 4 - k , and so we just need to consider k 4 < k 2 4 < 4 k k - 4 < \sqrt{k^{2} - 4} < 4 - k . The LHS follows from k 2 4 0 > k 4 \sqrt{k^{2} - 4} \ge 0 > k - 4 , since k < 4 k < 4 . And the RHS holds if and only if 2 k < 5 2 2 \le k < \frac{5}{2} , since by 4 k , k 2 4 0 4 - k, \sqrt{k^{2} - 4} \ge 0 and so we can square both sides to conclude 2 k < 5 2 2 \le k < \frac{5}{2} ( k 2 k \ge 2 ) is from the assumption.

Case 4 4 : 4 k 5 4 \le k \le 5

Since k 4 > ( k + 8 ) , k + 8 > 4 k k - 4 > -(k + 8), k + 8 > 4 - k , and so we again just need to consider k 4 < k 2 4 < 4 k k - 4 < \sqrt{k^{2} - 4} < 4 - k . But this time the RHS doesn't hold since k 2 4 0 4 k \sqrt{k^{2} - 4} \ge 0 \ge 4 - k , by k 4 k \ge 4 , which is a contradiction and thus we have no solution.

In conclusion, the solutions are 17 4 k 2 - \frac{17}{4} \le k \le -2 and 2 k < 5 2 2 \le k < \frac{5}{2} .

Now, it is now trivial to compute that 1000 p = 275 \lfloor 1000p \rfloor = \boxed {275} .

Titas Saha
May 20, 2014

the roots of the equation X^2+kX+1=0 is (-k+(k^2-4)^.5)/2 & (-k-(k^2-4)^.5)/2 the roots are between -2 and 4 . there fore we have k-4 less equal (k^2-4)^.5 less equal k+8 . and k-4 less equal -(k^2-4)^.5 less equal k+8 . now we can easily find out the solution of k for the real roots . the solution is -4.25 less equal k less equal -2 and 2 less equal k less equal 2.5 . hence the total length of interval where we can find k is (4.25-2) + (2.5-2) =2.75 the function f(x) defined in the closed interval -5 to 5 . the length of the interval is (5+5)=10. therefore the required probability is 2.75/10 ie 275/1000. hence the answer is (275/1000)*1000 = 275 .

=

f(x)=x2+kx+1. roots of this equation are (-k+/-√(k^2-4))/2. For least k value the root must be less than 4 thus (-k+/-√(k^2-4))/2=4 gives least k, similarly (-k+/-√(k^2-4))/2=-2 gives highest k ,solving those equations for k we get least value of k as -4.25 and highest value of k as 2.5. If the k value is between -2 and 2 then roots becomes imaginary ( k^2<4). So the actual k values are -4.25 to -2 and 2 to 2.5 .Thus constituting 2.75 (-2-(-4.25)+2.5-2=2.75). The overall interval is [−5,5] ,constituting 10(5-(-5)). Thus p=(2.75)/10 and ⌊1000p⌋=275

If we look at the problem from a graphical and calculus point of view........ The conditions just boils down to the quadratic function having a minima at x say m for which m lies between (-2,4) And also the vertex of the parabola to lie in the lower side of the coordinate plane . now this condition is given by f(m) < 0. Now f'(x) =0 for x= -k/2. Or m=-k/2 So the first condition is given by. f(-k/2) <0 . And also the sign of f(-2) and f(4) must be the same(in this case positive). So we get our second condition. Plug these together to find that k lies in (-4.25, -2) U (2, 2.5). Giving us the probability as 275/1000.

Jason Carlson
Nov 26, 2016

I think the answer may be slightly flawed. Correct me if I'm wrong on this because I'm curious. Since the question asks for the floor function of 1000p, I think the probability should drop to 274. But, then again, since we are essentially looking at an infinite / continuous range of values, .999999999999999999 repeating =1 so rip. ^Somebody explain this better lol/. I'm mixed with opinions. Help

Let Z be the roots of the quadratic f(x).

k ± k 2 4 2 \frac {-k \pm \sqrt{k^2 - 4}}{2}

We are asked to find P[-2 < Z < 4].

P [ 2 < Z < 4 ] = P [ 2 < k ± k 2 4 2 < 4 ] P[-2 < Z < 4] = P[-2 < \frac {-k \pm \sqrt{k^2 - 4}}{2} < 4] for -2<Z<-1 or 1<Z<4

P [ 2 < Z < 4 ] = P [ 2 < k k 2 4 2 < 1 ] + P [ 1 < k + k 2 4 2 < 4 ] P[-2 < Z < 4] = P[-2 < \frac {-k - \sqrt{k^2 - 4}}{2} < -1] + P[1 < \frac {-k + \sqrt{k^2 - 4}}{2} < 4]

We can solve K by solving the left side and the right side of the inequality separately.

ie. k k 2 4 2 < 1 \frac{-k - \sqrt{k^2 - 4}}{2} < -1

k + k 2 4 > 2 k + \sqrt{k^2 - 4} > 2

k 2 4 > k 2 4 k + 4 k^2 - 4 > k^2 - 4k + 4

k > 2 k > 2

After applying to all sides, we arrive at the following:

P [ 2 < Z < 4 ] = P [ 2 < K < 2.5 ] + P [ 4.25 < K < 2 ] P[-2 < Z < 4] = P[2 < K < 2.5] + P[-4.25 < K < -2]

The probability density function of the continuous uniform distribution is: f ( x ) = { 1 b a a x b 0 x < a o r x > b f(x) = \left\{ \begin{array}{ll} \frac{1}{b-a}\ & \quad a \leq x \leq b \\ 0 & \quad x < a\ or\ x > b \end{array} \right.

P [ 2 < K < 2.5 ] = 2 2.5 1 5 ( 5 ) d x = 0.05 P[2 < K < 2.5] = \int_2^{2.5} \frac{1}{5 - (-5)}\,\mathrm{d}x = 0.05

P [ 4.25 < K < 2 ] = 4.25 2 1 5 ( 5 ) d x = 0.225 P[-4.25 < K < -2] = \int_{-4.25}^{-2} \frac{1}{5 - (-5)}\,\mathrm{d}x = 0.225

P [ 2 < Z < 4 ] = 0.05 + 0.225 = 0.275 P[-2 < Z < 4] = 0.05 + 0.225 = 0.275

1000 0.275 = 275 \lfloor{1000*0.275}\rfloor = 275

Aldrian Obaja
May 20, 2014

First, we note that the roots are: k ± k 2 4 2 \frac{-k\pm\sqrt{k^2-4}}{2} . Since it must have two roots, k 2 4 0 k^2-4\geq 0 , therefore 2 < k < 2 -2 < k < 2 .

Next we note also that 2 k ± k 2 4 2 4 -2\leq\frac{-k\pm\sqrt{k^2-4}}{2}\leq 4 or 4 k ± k 2 4 8 -4\leq -k\pm\sqrt{k^2-4}\leq 8 . Since k 2 4 \sqrt{k^2-4} is positive, it is equivalent to saying 4 k k 2 4 -4\leq -k-\sqrt{k^2-4} and k + k 2 4 8 -k+\sqrt{k^2-4}\leq 8 .

For 4 k k 2 4 -4\leq -k-\sqrt{k^2-4} , which is equivalent to k 2 4 4 k \sqrt{k^2-4}\leq 4-k , since k 2 4 \sqrt{k^2-4} is positive, we can square both sides to get the final inequality k 2.5 k\leq 2.5 .

For k + k 2 4 8 -k+\sqrt{k^2-4}\leq 8 , which is equivalent to k 2 4 8 k \sqrt{k^2-4}\leq 8-k , since k 2 4 \sqrt{k^2-4} is positive, we can square both sides to get the final inequality 4.25 k -4.25\leq k .

Combining the above three inequality, we have: 4.25 k < 2 -4.25\leq k < -2 or 2 < k 2.5 2 < k \leq 2.5 , giving us the total interval length of 2.75 2.75 . So the probability is p = 2.75 10 = 0.275 p=\frac{2.75}{10}=0.275 , therefore 1000 p = 275 \lfloor 1000p\rfloor = 275

Elvin Gu
May 20, 2014

x = k ± k 2 4 2 x = \frac{-k \pm \sqrt{k^2-4}}{2} so we can find the maximum and minimum of k along the interval [-5,5]

4 = k + k 2 4 2 k = 5 2 4 = \frac{-k + \sqrt{k^2-4}}{2} \Rightarrow k = \frac{5}{2}

2 = k k 2 4 2 k = 17 4 -2 = \frac{-k - \sqrt{k^2-4}}{2} \Rightarrow k = -\frac{17}{4}

But to keep the roots real, k 2 4 0 k 2 \sqrt{k^2-4} \geq 0 \Rightarrow k \geq 2 or k 2 k \leq -2

Thus the valid values is 17 4 k 2 -\frac{17}{4} \leq k \leq -2 and 2 k 5 2 2 \leq k \leq \frac{5}{2}

2 + 17 4 + 5 2 2 = 11 4 11 4 100 = 275 -2+\frac{17}{4} + \frac{5}{2}-2 = \frac{11}{4} \Rightarrow \frac{11}{4}*100 = 275

And the floor of that is still 275 275

Tianxiao Liu
May 20, 2014

Determinant=k^2-4 \geq 0 Thus k \geq 2 or k \leq -2. As both roots are in between -2 and 4, -2 \leq \frac{-k \pm \sqrt{k^2-4}}{2} \leq 4. i.e. k-4 \leq \pm \sqrt{k^2-4} \leq k+8 Now we consider the 2 cases k \geq 2 and k \leq -2 separately. Case 1: If 2 \leq k \leq 5, then \pm \sqrt{k^2-4} \leq k+8 is always true for all k as the minimum value of k+8 (which is 10) is greater than the largest value of \pm \sqrt{k^2-4}. Similarly k-4 \leq \sqrt{k^2-4} is always true for all k in the interval of [2,5]. The only sub-case left is k-4 \leq -\sqrt{k^2-4}, which is 4-k \geq \sqrt{k^2-4} in other words. Notice that k /leq 4 in this case. Solving this inequality by taking square of both sides, we obtain the following solution 2 \leq k \leq 2.5. Case 2: If -5 \leq k \leq -2, similarly we can conclude that both k-4 \leq \pm \sqrt{k^2-4} and -\sqrt{k^2-4} \leq k+8 are always true for all k in the interval of [-5,-2]. The only sub-case left is \sqrt{k^2-4} \leq k+8. This can be solved by taking the square of both sides and the following solution is obtained: -4.25 \leq k \leq -2. Therefore p= \frac {(2.5-2) + (-2-(-4.25))}{5-(-5)}=0.275 Hence, 1000p=275

Tan Kin Aun
May 20, 2014

The roots of f ( x ) f(x) is k ± k 2 4 2 \frac{-k\pm \sqrt{k^2-4}}{2} . So, we must have 2 k + k 2 4 2 -2\leq \frac{-k+\sqrt{k^2-4}}{2} and 4 k k 2 4 2 4\geq \frac{-k- \sqrt{k^2-4}}{2} ,then we will have k 2.5 k\leq 2.5 and k 4.25 k\geq -4.25 . But we must have k 2 4 k^2\geq 4 to obtain real roots. So, k 2 k\geq 2 and k 2 k\leq -2 . The probability is ( 2.5 2 ) + [ 2 ( 4.25 ) ] 5 ( 5 ) = 2.75 10 \frac{(2.5-2)+[-2-(-4.25)]}{5-(-5)}=\frac{2.75}{10} . Therefore, 1000 p = 1000 2.75 10 = 275 \lfloor 1000p\rfloor=\lfloor1000\cdot \frac{2.75}{10}\rfloor =275 .Done.

0 pending reports

×

Problem Loading...

Note Loading...

Set Loading...